User avatar
 
ManhattanPrepLSAT1
Thanks Received: 1909
Atticus Finch
Atticus Finch
 
Posts: 2851
Joined: October 07th, 2009
 
This post thanked 4 times.
 
 

Re: Q22 - drug marketing

by ManhattanPrepLSAT1 Fri Dec 31, 1999 8:00 pm

The argument is designed to establish the conclusion that we should try a new marketing campaign. The evidence for why we should do this is that the product is not selling and that while not guaranteeing success, it's one way one might save the product.

The problem here is that the argument has failed to consider possible reasons why we should not try a new marketing campaign. The drug company manager has named two reasons why we might try a new campaign, but has not thoroughly weighed the pros and the cons.

To weaken this argument we need to find some potential drawback to a new marketing campaign. Answer choice (D) provides a potential negative impact of trying a new campaign, and so represents the correct answer.

What's tougher than seeing how answer choice (D) weakens the argument is seeing how the other answer choices do NOT!

(A) strengthens the argument that something should be done to help out the failing new product.
(B) doesn't tell us anything about the likelihood that trying a new marketing campaign will be successful.
(C) uses a qualifying trigger that is not going to happen. The evidence suggests there is a chance that the marketing campaign will succeed.
(E) is irrelevant to whether the impacts of a new marketing campaign for the new product would be positive or negative to the company.

Let me know if you still have questions on this one!


#officialexplanation
 
catie0128
Thanks Received: 0
Forum Guests
 
Posts: 11
Joined: May 17th, 2011
 
 
 

Q22 - Drug company manager: Our newest

by catie0128 Thu Jun 02, 2011 5:16 pm

Why D over the other answer choices?
 
sweliz
Thanks Received: 0
Vinny Gambini
Vinny Gambini
 
Posts: 4
Joined: April 09th, 2011
 
 
 

Q22 - Drug company manager: Our newest

by sweliz Sat Jun 04, 2011 7:17 pm

I didn't quite get this question either. Need some help! Thanks!
 
irenaj
Thanks Received: 0
Forum Guests
 
Posts: 18
Joined: August 31st, 2011
 
 
 

Re: Q22 - drug marketing

by irenaj Thu Sep 29, 2011 6:21 pm

If B says instead: "many new products fail whether not they are supported by new marketing campaigns", would it weaken the argument and be the correct answer?
User avatar
 
ManhattanPrepLSAT1
Thanks Received: 1909
Atticus Finch
Atticus Finch
 
Posts: 2851
Joined: October 07th, 2009
 
This post thanked 1 time.
 
 

Re: Q22 - drug marketing

by ManhattanPrepLSAT1 Sat Oct 01, 2011 3:29 pm

irenaj Wrote:If B says instead: "many new products fail whether not they are supported by new marketing campaigns", would it weaken the argument and be the correct answer?

Not really. The issue is that the stimulus acknowledges that the new marketing campaign will not ensure success - which means that answer choice (B) is consistent with the information already presented and would not weaken the argument. Remember, "many" simply means more than one. So the distinction between a "new marketing campaign" and a regular old "marketing campaign" is not very important here. Though on other questions such a distinction is very important. For instance in PT30, S2, Q16 you need this distinction to eliminate a tempting but incorrect answer choice.

Good question!
 
yifanfeng
Thanks Received: 3
Forum Guests
 
Posts: 2
Joined: October 01st, 2012
 
This post thanked 3 times.
 
 

Re: Q22 - Drug company manager: Our newest

by yifanfeng Mon Oct 01, 2012 7:45 pm

My problem with answer D is that it appeals to the principle of maximizing the company's overall positions rather than the sales figure of the newest product. The stimulus is certainly ambiguous about what it means by "success" and it may be argued that a drug company manager would naturally want to maximize profit, but the criteria for the decision to begin a new marketing campaign is more accurately interpreted, in my opinion, as the success of selling the newest product rather than the bottom line. In previous questions of this type (e.g. PT52S3Q16), answers that do not challenge a premise in the stimulus directly or one of its necessary assumptions still must appeal to the principle est. in the question, rather than an assumed principle.

I chose answer E with the problematic assumption that, because the company has been successful overall, it's not brand recognition that's the problem. Therefore, an additional marketing campaign will do no good. But if I am right about answer D, E would be problematic only because its assumption is more unbelievable than that of D, a tenuous basis for answering the question.
 
amiraly
Thanks Received: 1
Forum Guests
 
Posts: 3
Joined: June 11th, 2013
 
This post thanked 1 time.
 
 

Re: Q22 - Drug company manager: Our newest

by amiraly Fri Jun 06, 2014 1:55 pm

The question stimulus says which if following is true though, so technically, (C) if it were true, in that there was NO CHANCE to succeed but the manager wrongfully or naively thought there was a slight chance than that would definitely weaken the argument. It seems the argument was about the product, now the company's overall success. The question stimulus is what threw me off by saying which if following were true, would waken, even though the core states "no guarantee" , an answer that says "theres no chance" in my opinion, at least, would most definitely weaken it. But i certainly understand why D is correct. Just another annoying LSAT question that is inconsistent.
 
mornincounselor
Thanks Received: 4
Elle Woods
Elle Woods
 
Posts: 54
Joined: June 25th, 2014
 
 
 

Re: Q22 - Drug company manager: Our newest

by mornincounselor Tue Sep 02, 2014 1:14 am

What I found difficult about this problem was I didn't recognize what the success of the business overall had to due with saving the single product.

Essentially, I forgot about the bit after the final comma.

If the conclusion of the argument was "while this would not guarantee success it is one chance to save the product" then our only means of weakening the argument would be to show why doing the campaign would not in fact save the product. None of the choices would really do this but (B) would be the only one related at all.

Once we take that final clause, the conclusion becomes "we should do this campaign" then that opens up the possibility of weakening the argument by establishing the possibility that the campaign would result in a net negative for the company.

That's what (D) does.